Monday, November 15, 2010

Some Problems in Number Theory

1.  Show that if n is a composite number then n divides (n-1)!

2.  Find the last two digits of the number 5^5^10000.

3.  Is log_10 2 rational or irrational?

4. How many zeros are there at the end of 100!

5. Can the number 111111111....1  (a string of n ones, where n >1) be a perfect  square?

Monday, November 1, 2010

An interesting geometry problem

This is not a Putnam problem, but it could be one. I have been struggling to prove this. If you can try and
let me know if have any success, that would be great.

Consider an acute angled triangle ABC. (This is a traingle in which all 3 angles are less than 90 degrees. ) Let PQR be a light triangle inside ABC. That is, it is a triangle inscribed by a light ray that starts on a point P on AB and then hits a point Q on BC, gets reflected and hits a point R on CA, and finally returns to a starting position A after a second reflection. These rays satisfy the usual property (angle of incidence = angle of reflection) of light at each of the points P, Q, and R.  Show that PRQ is an altitude triangle. That is, P, Q and R are the feet of the perpendiculars drawn from C, B and A (respectively) to to their opposite sides.